Two rods, Two hinges.

The setup shown consists of two identical rods (mass m) hinged together at the ends.
This setup is held in the position shown.
Now we start applying a constant force F to the free end as shown
Find the initial angular acceleration of the rods.

39 Answers

1
Philip Calvert ·

qwerty : justify P being thre ICR.

1
Philip Calvert ·

by the way i was restlessly waiting for some responses by some experts in this thread...

still waiting though...

1
Philip Calvert ·

Ok try doing it by energy method..

Write total enery = cons and differentiate ..

1
Unicorn--- Extinct!! ·

Philip plz post ur soluton. And do it by force method.

1
Philip Calvert ·

Anyone interested ?

1
Sonne ·

qwerty is solving

23
qwerty ·

o.O

lol

23
qwerty ·

i wil giv it a try though

\tau _{c}=I_{c}\alpha
(F-F_{x})\frac{l}{2}sin\theta +F_{y}lcos\theta =I_{c}\alpha............................(1)

F-F_{x}=2ma_{x}

2mg+F_{y}=2ma_{y}

a_{c}=a_{x}(\hat{i})-a_{y}(j)

now Point P is ICR .

Hence COM performs pure rotation abt pt P.

thus ,

i.e,\vec{a_{c}}=PCsinA\alpha (\hat{i})-PCcosA(\hat{j})

i.e,\vec{a_{c}}=\frac{l}{2}sin\theta \alpha (\hat{i})-lcos\theta \alpha (\hat{j})

hence
F-F_{x}=2ma_{x}=2m\frac{l}{2}sin\theta \alpha =ml\alpha sin\theta

2mg+F_{y}=2ma_{y}=2ml\alpha cos\theta

putting the values of F-Fx and Fy in torque eqn

(ml\alpha sin\theta )\frac{l}{2}sin\theta +(2ml\alpha cos\theta -2mg)lcos\theta =I_{c}\alpha

\frac{ml^{2}sin^{2}\theta \alpha }{2} +2ml^{2}cos^{2}\theta \alpha -I_{c}\alpha =2mglcos\theta

\alpha = \frac{2mglcos\theta }{\frac{ml^{2}sin^{2}\theta }{2} +2ml^{2}cos^{2}\theta -I_{c}}

I_{c}=\frac{2ml^{2}}{12}+\frac{2ml^{2}cos^{2}\theta }{4}

\alpha = \frac{2mglcos\theta }{\frac{ml^{2}sin^{2}\theta }{2} +2ml^{2}cos^{2}\theta -(\frac{2ml^{2}}{12}+\frac{2ml^{2}cos^{2}\theta }{4}) }

\alpha = \frac{6gcos\theta }{l(3cos^{2}\theta +1) }

23
qwerty ·

it can be done even widout ICR

system rotates abt com with alpha

so ap = ac/p+ap=CPsinA(i) - CPcosA(j) + ax(i) -ay(j) = 0

hence ax=CPsinA , ay=CPcosA

23
qwerty ·

correction in last post ,
ap = ac/p+ap=CPsinA(-i) + CPcosA(j) + ax(i) -ay(j) = 0

1
Philip Calvert ·

@eureka : that was a reply to the general reader
to give them a complete picture lest they should commit the same mistake as virang1 did.
basically what i was pointing out was since there are both these forces present the choice of a point about which we can apply torque eqn is difficult to make.

.. specifically for your solution the torque due to gravity will definitely be zero.

23
qwerty ·

it can be done widout ICR .........

the hinge at P is fixed , so its accn shud be zero ,see #29,30

23
qwerty ·

no one?

1
ujjwalkalra kalra ·

philip plz post the solution

23
qwerty ·

so no one is gona reply ? not even mentors ?

71
Vivek @ Born this Way ·

Nice for reviving the topic!

49
Subhomoy Bakshi ·

the above figure illustrates the initial condition...t=0

now at any arbit time t let the angle be β

now look at time t the angular velocity of both the rods will be the same by symmetry and the rod towards right will have traslational motion apart from its rotational motion...

thus we get,

mgl(sinθ-sinβ)+2Fl(cosβ-cosθ)=12Iω2+12Iω2+12mv2

v=l.ω (i.e. vel of top most point of leftward rod)

gives,

mgl(sinθ-sinβ)+2Fl(cosβ-cosθ)=Iω2+12ml2ω2=13ml2ω2+12ml2ω2=56ml2ω2

i.e. mgl(sinθ-sinβ)+2Fl(cosβ-cosθ)=56ml2ω2

differentiating we get,

56ml2.2ωα=-(mglcosβ)ω-(2Flsinβ)ω

gives α = - 6(mgcosβ+2Fsinβ)5ml

i hope that is the answer!!!

23
qwerty ·

subho has done it correct

energy method se hi karna chahiye like subho

btw subho nice solution yaar

never thought abt doing it the energy way

btw subho pls point out da mistake in my solution

23
qwerty ·

i got my mistake ,the 2 rod system isnt rigid , so some eqns go wrng

49
Subhomoy Bakshi ·

right....

the first rod is ok but to the right rod the COM has Accn as well as angular accn and these two things are related as a=Lα

also another forward accn of the right rod a'=α(L/2)

i think u missed on this!!!

1
Philip Calvert ·

and also you did not notice that |α| should be same for both :( even though i mentioned it in post #13 itself

1
Philip Calvert ·

the hinge you see in the middle is not fixed

24
eureka123 ·

kk......... i thought both are fixed[3]

62
Lokesh Verma ·

no one?

19
Debotosh.. ·

i sense that this sum will involve the basic definition of angular velocity .... dθ /dt !
....or is it ....?

1
Philip Calvert ·

First of all notice that the magnitude of α will be the same for both...

I am not too keen to give any further hint because that hint will be the solution itself.

maybe after someone has atleast tried

11
virang1 Jhaveri ·

First resolve the force F one perpendicular to the rod and another along the rod.

Therefore Fp = Fsinθ
Fa=Fcosθ
Now Fa will produce x acceleration to the rod in its direction. Therefore
Fa = 2mx
x=Fa/2m
Now this mx will be the force that will provide the torque to the left hand side rod.

Therefore now angular acceleration
For the left one
mxl = ml2/3*α
α=3x/l=3Fa/2ml=3Fcosθ/2ml

For the right one
FpL = mL2/3*α
α=3Fp/mL
α = 3Fsinθ/ml

1
Philip Calvert ·

no.......

you seem to have left out all the forces except F.....

! ? [11]

how do you justify that

11
virang1 Jhaveri ·

I missed gravity.

1
Philip Calvert ·

@virang1 : did not want to discourage the only person who tried....

but honestly u are not even close.

Your Answer

Close [X]